You are on page 1of 19

CHUYÊN ĐỀ

HÀM vp VÀ ỨNG DỤNG


Phạm Xuân Thịnh, trường THPT chuyên Hạ Long, Quảng Ninh 1

Tóm tắt nội dung

Chuyên đề này trình bày về cấp của một số nguyên, căn nguyên thủy và ứng dụng của
chúng trong các bài toán số học. Trong bài viết, chúng tôi đưa ra những chứng minh chi
tiết cho các định lí và những áp dụng trực tiếp của chúng. Ngoài ra, chúng tôi cũng đưa
ra một chứng minh mới, chỉ bằng kiến thức THCS cho định lí Zsigmondy.

Ký hiệu và quy ước

N : tập các số tự nhiên


N∗ : tập các số nguyên dương
Z : tập các số nguyên
P : tập các số nguyên tố
vp (a) : số mũ cao nhất của số nguyên tố p trong biểu diễn chính tắc của số nguyên a.

1 Hàm vp và các tính chất

Định nghĩa 1. Cho số nguyên dương n và p là một số nguyên tố. Khi đó tồn tại số nguyên tự
nhiên m sao cho pm | n và pm+1 ∤ n. Số m như vậy được gọi là số mũ đúng của p trong phân
tích tiêu chuẩn của n và được kí hiệu bởi vp (n).
Tính chất cơ bản. Với a, b là các số nguyên dương bất kì và p là một số nguyên tố. Khi ấy,
ta có

i) vp (ab) = vp (a) + vp (b).

ii) vp (an ) = n.vp (a).

iii) vp (a + b) ≥ min {vp (a), vp (b)} . Dấu "=" xảy ra khi và chỉ khi vp (a) 6= vp (b).

Việc chứng minh các tính chất này nói chung là dễ.
1
Các trao đổi với tác giả về bài viết này có thể liên hệ qua email : phamxthinh@gmail.com
2 Phạm Xuân Thịnh

Định lý 1 (Legendre). Cho số nguyên dương n. Khi đó, ta có


+∞      
X n n n
vp (n!) = = + 2 + ...
k=1
pk p p

Chứng minh.

Bổ đề (Bổ đề nâng lũy thừa).


Chú ý.

Sau đây là một số ví dụ minh họa


Ví dụ 1 (Taiwan TST 2016). Tìm các số nguyên dương a > b thỏa mãn (a − b)ab = ab ba .

Giải. Nếu b = 1, thì (a − 1)a = a. Dễ thấy điều này không thể xảy ra. Do đó, a > b ≥ 2. Khi
ấy nếu p là một ước nguyên tố bất kì của b, thì p | a. Gọi p là một ước nguyên tố của b và đặt
x = vp (a), y = vp (b), x, y ∈ N∗ , thì từ giả thiết ta có

b.x + a.y = vp (ab ba ) = vp (a − b)ab ≥ ab. min {x, y} .




Chú ý rằng a ≥ px ≥ p.x và b ≥ py ≥ p.y. Do đó,


x y 2
min {x, y} ≤ + ≤ ≤ 1.
a b p

Xảy ra các trường hợp sau

i) Nếu x ≥ y, thì y = 1. Khi ấy, ab ≤ a + b.x suy ra b.x ≥ a(b − 1) ≥ 2x(b − 1) hay b ≤ 2. Do
đó, b = 2 và a = 2x. Nhưng a ≥ 2x ≥ 2x nên x = 1 hoặc x = 2, tức là a = 2 hoặc a = 4.
Vì a > 2 nên a = 4. Thử lại ta thấy a = 4, b = 2 thỏa mãn đề bài.

ii) Nếu x < y, thì x = 1 và ta có ab ≥ a.y + b. Từ đây suy ra a.y ≥ b(a − 1) ≥ 2y(a − 1). Do
đó, a ≤ 2. Vô lí, vì a > 2.

Vậy (a, b) = (4, 2) là nghiệm duy nhất của bài toán.


Ví dụ 2. Cho a, b, c là các số nguyên dương thỏa mãn c(ca + 1)2 = (2c + b)(3c + b). Chứng
minh rằng c là một số chính phương.

Giải. Gọi p là ước nguyên tố bất kì của c, thì gcd(p, ca + 1) = 1. Từ giả thiết ta thấy c | b2 ,
suy ra vp (v) ≤ 2vp (b). Nếu vp (b) ≥ vp (c), thì do

vp (2c + b) ≥ min {vp (2c), vp (b)} ≥ vp (c) và vp (3c + b) ≥ min {vp (3c), vp (b)} ≥ vp (c)

nên vp (c) = vp (2c + b) + vp (3c + b) ≥ 2vp (c). Vô lí ! Do đó, vp (b) < vp (c). Khi ấy,

vp (c) = vp (2c + b) + vp (3c + b) ≥ 2vp (b).

Từ các lí luận trên ta thu được vp (c) = 2vp (b). Vậy c là số chính phương.
Cấp của số nguyên, căn nguyên thủy và ứng dụng 3

Ví dụ 3 (Poland 2016). Cho k, n là các số nguyên dương lẻ lớn hơn 1. Chứng minh rằng nếu
tồn tại số tự nhiên a thỏa mãn k | 2a + 1 và n | 2a − 1 thì không tồn tại số tự nhiên b sao cho
k | 2b − 1 và n | 2b + 1.

Giải. Giả sử tồn tại số tự nhiên b sao cho k | 2b − 1 và n | 2b + 1. Đặt m = ordk (2), thì
ta có m | b, m | 2a và m ∤ a. Từ m | 2a và m ∤ a, ta suy ra v2 (a) < v2 (m) ≤ v2 (a) + 1 hay
v2 (m) = v2 (a) + 1. Mà v2 (m) ≤ v2 (b) nên v2 (a) + 1 ≤ v2 (b). Chứng minh tương tự ta cũng có
v2 (b) + 1 ≤ v2 (a). Mâu thuẫn ! Vậy điều giả sử là sai, tức là ta có điều phải chứng minh.
Ví dụ 4. Chứng minh rằng với mọi số nguyên n > 1, thì phương trình sau không có nghiệm
hữu tỉ
xn xn−1 x2
+ +···+ + x + 1 = 0.
n! (n − 1)! 2!

Giải. Phương trình đã cho tương đương với


xn + nxn−1 + n(n − 1)xn−2 + · · · + n(n − 1) . . . 4.3x2 + n!x + n! = 0.
a
Giả sử phương trình ban đầu có nghiệm hữu tỉ x = với a, b ∈ Z, b > 0 và gcd(a, b) = 1. Khi
b
ấy b | an suy ra b = 1. Do đó, phương trình đã cho có nghiệm nguyên. Vì n > 1 nên n có ước
nguyên tố p. Từ
an + nan−1 + n(n − 1)an−2 + · · · + n(n − 1) . . . 4.3a2 + n!a + n! = 0 (1)
suy ra p | a. Dễ thấy vp (k!) < k nên
 
n! k
vp a = vp (n!) + kvp (a) − vp (k!) > vp (n!) + k (vp (a) − 1) ≥ vp (n!).
k!
Cũng vì p | a nên vp (an ) = nvp (a) ≥ n > vp (n!). Do đó, (1) không thể xảy ra. Vậy điều giả sử
là sai, tức là phương trình đã cho không có nghiệm hữu tỉ.
Ví dụ 5 (IMOSL 2015). Tìm tất cả các số nguyên dương M sao cho dãy số (an ) xác định bởi
1
a0 = M + , an+1 = an .[an ], n ≥ 0 chứa ít nhất một số nguyên.
2

M
/ N, ∀n ≥ 0. Vì a1 = M 2 +
Giải. Giả sử an ∈ nên M phải là số lẻ. Chú ý là nếu đặt
2
2M 2 + M − 1
M1 = , thì M1 ∈ N và
2
2M 2 + M − 1 1 1
a1 = + = M1 + .
2 2 2
Vì a2 ∈
/ N nên M1 là số lẻ. Cứ tiếp tục như vậy, ta thấy dãy số (Mn ) xác định bởi
2Mn2 + Mn − 1
M0 = M và Mn+1 = , n≥0
2
2
chứa toàn số lẻ. Xét số nguyên dương n bất kì, thì do Mn ≡ 1 (mod 2) nên 2Mn−1 +Mn−1 −1 ≡ 2
2
(mod 4). Từ đây suy ra Mn−1 ≡ 1 (mod 4). Khi đó, 2Mn−2 +Mn−2 −1 ≡ 2 (mod 8) hay Mn−2 ≡
1 (mod 8). Cứ tiếp tục như vậy, ta sẽ được M = M0 ≡ 1 (mod 2n+1 ). Do đó, v2 (M − 1) ≥ n+1.
Cho n → +∞ ta suy ra M = 1.
3
Dễ thấy M = 1, thì an = / N, ∀n ≥ 0. Vậy mọi M ≥ 2 đều thỏa mãn đề bài.

2
4 Phạm Xuân Thịnh

Ví dụ 6. Cho p là số nguyên tố. Tìm tất cả các số nguyên n > 1 sao cho
gcd p, Cn1 = gcd p, Cn2 = · · · = gcd p, Cnn−1 = 1.
  

Giải. Giả sử n > 1 là một số nguyên thỏa mãn đề bài. Dễ thấy gcd(p, n) = 1. Do đó, tồn tại
các số tự nhiên m, s, r với 0 < m < p, 0 < r < ps sao cho n = ps .m + r. Ta có
(ps .m + r) (ps .m + r − 1) . . . (ps .m)
Cpr+1
s .m+r = .
(r + 1)!
Chú ý là vp (ps .m + k) = vp (k), ∀k = 1, r. Từ đây suy ra
vp Cpr+1

s .m+r = s − vp (r + 1).
.
Vì gcd p, Cpr+1 = 1 nên vp Cpr+1 = 0 hay vp (r + 1) = s, tức là r + 1 .. ps . Nhưng
 
s .m+r s .m+r

0 < r + 1 ≤ ps nên suy ra r + 1 = ps hay r = ps − 1. Do đó, n = ps .u − 1 với s ∈ N, u = 2, p.



Bây giờ giả sử n = ps .u − 1 với s ∈ N, u = 2, p, ta sẽ chứng minh vp Cpks.u−1 = 0, ∀k =
1, ps .u − 2. Thật vậy, ta có
(ps .u − 1) (ps .u − 2) . . . (ps .u − k)
Cpks.u−1 = .
k!
Với mọi i ∈ {1; 2; . . . ; k} , thì i < ps+1 suy ra vp (i) < s+1. Nếu vp (i) < s, thì vp (ps .u − i) = vp (i),
còn nếu vp (i) = s, thì i = ps .v với (1 ≤ v < u).  Khi ấy, p .u − i =k (u −
s
 v).p . Do 1 ≤ u − v < p
s

nên vp (p .u − i) = vp (i). Như vậy vp Cps .u−1 = 0 hay gcd p, Cps.u−1 = 1, ∀k = 1, ps .u − 2.


s k

Tóm lại, n = ps .u − 1 với s ∈ N, u = 2, p là tất cả các số nguyên dương thỏa mãn đề bài.
Ví dụ 7 (China TST 2016). Cho các số nguyên c, t ≥ 2. Xét dãy số (an ) xác định bởi
a1 = c, an+1 = atn + c, ∀n ≥ 1. Chứng minh rằng với mọi n ≥ 2 luôn tồn tại số nguyên tố p
sao cho p | an và p ∤ ai , ∀i = 1, n − 1.

Giải. Giả sử ngược lại là tồn tại số nguyên dương n ≥ 2 sao cho mọi ước nguyên tố p của an ,
n−1
thì p | ai . Xét p là một ước nguyên tố bất kì của an , thì tồn tại m < n sao cho p | am . Khi
Q
i=1
đó, ta có
am+1 = atm + c ≡ c (mod p) hay am+1 ≡ a1 (mod p).
Bằng quy nạp ta dễ dàng chứng minh được am+k ≡ ak (mod p), ∀k ≥ 1. Hệ quả là an ≡ an−m
(mod p) hay p | an−m . Do đó, theo thuật toán Euclide, ta suy ra p | agcd(m,n) . Kí hiệu d =
gcd(m, n), thì p | ad . Đặt u = vp (ad ), thì
ad+1 = atd + c ≡ c (mod put ) hay ad+1 ≡ a1 (mod put )
và bằng quy nạp ta chứng minh được ad+k ≡ ak (mod put ), ∀k ≥ 1. Từ đây suy raakd ≡ ad
n−1
(mod put ), ∀k ≥ 1. Do đó, an ≡ ad (mod put ) và vì thế ta có vp (an ) = vp (ad ) ≤ vp
Q
ai .
i=1
n−1
Mà p là ước nguyên tố bất kì của an nên an ≤ ai . Mặt khác, ta lại có an = atn−1 + c > atn−1 .
Q
i=1
Từ đánh giá này ta suy ra
n−1
Y
t−1
an > (an−1 .an−2 . . . a1 ) a1 > ai .
i=1
Cấp của số nguyên, căn nguyên thủy và ứng dụng 5

Điều mâu thuẫn này chứng tỏ điều giả sử là sai, tức là bài toán được chứng minh là đúng.
Ví dụ 8 (IMOSL 2014). Cho số nguyên c ≥ 1 và dãy số nguyên dương (an ) xác định bởi
(
a1 = c
an+1 = a3n − 4ca2n + 5c2 an + c, ∀n ≥ 1.

Chứng minh rằng với mọi n ≥ 2 luôn tồn tại số nguyên tố p sao cho p | an và p ∤ pi , ∀i = 1, n − 1.

Giải. Với mỗi n ≥ 2, ta có an+1 = a3n − 4ca2n + 5c2 an + c ≡ 5c2 an + c (mod a2n ) suy ra

an+2 = a3n+1 − 4ca2n+1 + 5c2 an+1 + c


3 2
≡ 5c2 an + c − 4c 5c2 an + c + 5c2 5c2 an + c + c (mod a2n )


≡ 2c3 + c = a2 (mod a2n ).

Bằng quy nạp, ta dễ dàng chứng minh được

an+k ≡ ak (mod a2n ), ∀k ≥ 2. (∗)


n−1
Giả sử tồn tại n ≥ 2 sao cho với mọi ước nguyên tố p của an ta đều có p | ai . Xét ước
Q
i=1
nguyên tố p bất kì của an , thì tồn tại m < n sao cho p | am . Khi đó, từ (∗) ta suy ra an ≡ an−m
(mod a2m ) hay p | an−m . Theo thuật toán Euclide, ta suy ra p | ah với h = gcd(n, m). Đặt
n = h.x và vp (ah ) = t, thì từ (∗) ta có

an = ah.x ≡ ah(x−1) ≡ ah(x−2) ≡ · · · ≡ ah (mod a2h ),


n−1 
suy ra vp (an − ah ) ≥ 2t. Do đó, vp (an ) = vp (ah ) ≤ vp ai . Mà p là ước nguyên tố bất kì
Q
i=1
n−1 n−1
của an nên an | ai . Hệ quả là an ≤ ai . Mặt khác, bằng quy nạp ta dễ dàng chứng minh
Q Q
i=1 i=1
được an − 2c > an−1 , ∀n ≥ 2. Do đó,

an = an−1 (an−1 − 2c)2 + c2 an−1 + c > an−1 (an−1 − 2c)2 > an−1 a2n−2 .

Từ đây suy ra
n−1
Y
an−1 a2n−2 an−3 a2n−4

an > = an−1 an−2 > ··· > ai .
i=1

Điều mâu thuẫn này chứng tỏ giả sử ở trên của ta là sai, tức là phải tồn tại ước nguyên tố p
n−1
của an sao cho p ∤
Q
ai .
i=1

.
Ví dụ 9. Cho hàm số f : N∗ → N∗ thỏa mãn f (a) − f (b) .. a − b, ∀a, b ∈ N∗ và a 6= b. Đặt
P = {p ∈ P | ∃a ∈ N∗ : p | f (a)} . Chứng minh rằng tập P là vô hạn nếu f khác hằng.

Giải.
6 Phạm Xuân Thịnh

Ví dụ 10. Cho số nguyên k > 1. Chứng minh rằng hàm f : N∗ → Z xác định bởi f (n) = nk −n!
là đơn ánh.

Giải. Trước hết ta có nhận xét sau


Nhận xét. Nếu các số nguyên dương m, n thỏa mãn m > n và m > 2k, thì f (n) > f (m).

Thật vậy, ta có

m! − n! = (m − 1)(m − 1)! + (m − 1)! − n! ≥ (m − 1)(m − 1)!.

Mà (m − i)(i + 1) > m, ∀i = 1, k nên

(m − 1)(m − 1)! = (m − 1)2 (m − 2)(m − 3) . . . (m − (k − 1)) .(m − k). (m − (k + 1))!


≥ (m − 1)2 (m − 2)(m − 3) . . . (m − (k − 1)) .2.k!
k−1
Y
= 2(m − 1) (m − i)(i + 1) > mk > mk − nk .
i=1

Từ đó suy ra nk − n! > mk − m! hay f (n) > f (m). Vậy nhận xét được chứng minh.
Trở lại bài toán ta giả sử f không phải là một đơn ánh, tức là tồn tại các số nguyên dương
m > n sao cho f (m) = f (n) hay

mk − nk = m! − n! (1)

Nếu n = 1, thì mk = m!. Vì k > 1 nên m > 2. Mà gcd(m, m − 1) = 1 và m − 1 > 1 nên điều
này không thể xảy ra. Do đó, n > 1. Gọi p là ước nguyên tố bất kì của n, thì từ (1) suy ra
p | m. Xảy ra 2 trường hợp sau

• Nếu n có ước nguyên tố p > 2, thì

vp (n!) = vp (m! − n!) = vp mk − nk ≥ k.




Sử dụng công thức Legendre, ta có


+∞   +∞
X n X n n
vp (n!) = ≤ < .
i=1
pi i=1
p i p − 1

Suy ra n > (p − 1)k ≥ 2k. Do đó, m > n > 2k. Theo nhận xét trên thì f (n) > f (m).
Mâu thuẫn !

• Nếu n = 2u , u ∈ N∗ , thì đặt v = v2 (m), ta có

n > v2 (n!) = v2 (m! − n!) = v2 mk − nk ≥ k. min {u, v} .




Nói riêng, n > k ≥ 2 nên u ≥ 2. Nếu v ≥ 2, thì m > n > 2k. Khi ấy, f (n) > f (m). Mâu
thuẫn ! Do đó, v = 1 hay m = 2.M với M là một số lẻ. Vì M > 1 nên M có ước nguyên
tố là q. Nếu q ≤ n, thì từ (1) suy ra q | n hay q = 2. Mâu thuẫn !. Thành thử, q > n suy
ra m > 2n > 2k. Từ đây lại suy ra f (n) > f (m). Mâu thuẫn !.
Cấp của số nguyên, căn nguyên thủy và ứng dụng 7

Từ những lập luận trên ta thấy điều giả sử là sai hay f phải là một đơn ánh.

Ví dụ 11. Cho x, y là các số nguyên dương thỏa mãn (x, y) = 1 và xy > 1. Chứng minh rằng
tồn tại vô số nguyên tố p sao cho vp (xp−1 − y p−1) là số lẻ.

Giải.

m+k
Ví dụ 12 (IMOSL). Cho các số nguyên dương m > n. Đặt xk = , k = 1, n + 1. Chứng
n+k
minh rằng nếu các số x1 , x2 , . . . , xn+1 đều nguyên thì x1 x2 . . . xn+1 − 1 chia hết cho một số
nguyên tố lẻ.

m−n
Giải. Đặt A = x1 x2 . . . xn+1 − 1. Vì xk = + 1 nên xk ∈ Z, ∀k = 1, n + 1 khi và chỉ khi
n+k
n + k | m − n, ∀k = 1, n + 1. Điều này xảy ra khi và chỉ khi lcm(n + 1, n + 2, . . . , 2n + 1) | m − n.
Đặt M = lcm(n + 1, n + 2, . . . , 2n + 1) và m − n = M.h thì ta có
     
M M M
A= .h + 1 . .h + 1 . . . .h + 1 − 1.
n+1 n+2 2n + 1

Gọi u là số tự nhiên thỏa mãn 2u < 2n + 1 < 2u+1 , thì n < 2u < 2n + 1, tức là 2u ∈
{n + 1, n + 2, . . . , 2n + 1} . Nếu có 2 số k, l (1 ≤ l < k ≤ n + 1) sao cho n + k ≡ n + l (mod 2u ),
thì 2u | k − l. Nhưng điều này không thể xảy ra vì 0 < k − l ≤ n < 2u . Do đó, u = v2 (M) và
trong tập {n + 1, n + 2, . . . , 2n + 1} chỉ có đúng một số chia hết cho 2u . Hệ quả là trong các số
M M
(k = 1, n + 1) có đúng một số là số lẻ, chẳng hạn là số lẻ. Bây giờ nếu A không
n+k n + k0
có ước nguyên tố lẻ thì A = 2v , v ∈ N. Thế thì h phải là số chẵn. Đặt v2 (h) = t, thì 1 < t < v
và ta có
M
A≡ h ≡ 2t (mod 2t+1 ).
n + k0
Điều này là vô lí, vì A ≡ 0 (mod 2t+1 ). Vậy A phải có ít nhất một ước nguyên tố lẻ và bài toán
được chứng minh xong.

Ví dụ 13 (China TST 2016). Với mỗi số nguyên dương m = 2k .t với k là số tự nhiên và t


là số nguyên dương lẻ, đặt f (m) = t1−k . Chứng minh rằng với mọi số nguyên dương n và số
n
nguyên dương lẻ a ≤ n thì f (m) chia hết cho a.
Q
m=1

n
Giải. Bài toán tương đương với việc chứng minh f (m) chia hết cho pα , với p là số nguyên
Q
m=1
tố lẻ bất kì và α là số nguyên
 dương thỏa mãn pα ≤ n. Điều này lại tương đương với việc
n
chứng minh vp f (m) ≥ α. Xét số nguyên tố p và số tự nhiên α như vậy, thì có đúng
Q
  m=1
n
số nguyên dương không vượt quá n và chia hết cho pi . Gọi u, v là các số tự nhiên thỏa
pi
mãn 2u ≤ n < 2u+1 và pv ≤ n < pv+1 , thì v ≥ α. Khi đó,
8 Phạm Xuân Thịnh

v  
X n
• k pi
là tổng của tất cả các số vp (m) với m = 2x .t ≤ n thỏa mãn p | m và
i=1
2
v2 (m) = x ≥ k.
v  
X n
• k+1 pi
là tổng của tất cả các số vp (m) với m = 2x .t ≤ n thỏa mãn p | m và
i=1
2
v2 (m) = x ≥ k + 1.

v   X v  
X n n
Do đó, k pi
− k+1 pi
là tổng của tất cả các số vp (m) với m = 2k .t ≤ n thỏa mãn
i=1
2 i=1
2
p | m và v2 (m) = k. Khi ấy, ta có
n
! u v   X v  !
Y X X n n
vp f (m) = (1 − k) −
m=1 k=0 i=1
2 k pi i=1
2k+1pi
v   u X v   X v   X u  !
X n X n n n
= − = − .
i=1
pi k=1 i=1
2 k pi i=1
pi k=1
2 k pi

Để ý rằng với mỗi i = 1, v, ta có


  X u   u
!   
n n X 1 n 1 n
− ≥ 1− = u i >0
pi k=1
2 k pi k=1
2k pi 2 p
   
n u n
suy ra i − ≥ 1. Do đó,
P
p k i
k=1 2 p

n
! v   X u  !
Y X n n
vp f (m) = i
− k pi
≥ v ≥ α.
m=1 i=1
p 2
k=1

Vậy bài toán được giải quyết xong.


Ví dụ 14. Với mỗi số nguyên dương n, tìm gcd (Cn1 , Cn2 , . . . , Cnn−1) .

Giải. Đặt d = gcd (Cn1 , Cn2 , . . . , Cnn−1) . Nếu d > 1, thì d có ước nguyên tố là p. Xảy ra 2
trường hợp sau

i) Nếu n có ít nhất 2 ước nguyên tố, thì tồn tại số nguyên dương k thỏa mãn pk < n < pk+1 .

 k
vp Cnp = vp (n!) − vp pk ! − vp (n − pk )!
 

k    k 
n − pk

X n p
= i
− i −
i=1
p p pi
k     
X n k−i n k−i
= i
−p − i +p =0
i=1
p p
k
nên p ∤ Cnp . Điều mâu thuẫn này chứng tỏ d = 1 hay
gcd Cn1 , Cn2 , . . . , Cnn−1 = 1.

Cấp của số nguyên, căn nguyên thủy và ứng dụng 9

ii) Nếu n chỉ có một ước nguyên tố, thì do p | Cn1 = n nên n = pm , m ∈ N∗ . Khi đó dễ dàng
chứng minh được p | Cnk , ∀k = 1, n − 1. Ngoài ra, do
 m−1 
vp Cppm = vp (pm !) − vp pm−1 ! − vp (pm − pm−1 )!
 

m  m   m−1   m
p − pm−1

X p p
= − −
i=1
pi pi pi
m−1
X
pm−i − pm−i−1 − pm−i − pm−i−1
 
= +1=1
i=1

nên gcd (Cn1 , Cn2 , . . . , Cnn−1 ) = p.

Ví dụ 15. Chứng minh rằng với mọi số nguyên dương n, ta có


 lcm(1, 2, 3, . . . , n, n + 1)
lcm Cn0 , Cn1 , Cn2 , . . . , Cnn = .
n+1

Giải. Đặt A = (n + 1)lcm (Cn0 , Cn1 , Cn2 , . . . , Cnn ) và B = lcm(1, 2, 3, . . . , n, n + 1), thì do (n +
k+1
1)Cnk = (k + 1)Cn+1 , ∀k = 0, n nên

A = lcm (n + 1)Cn0 , (n + 1)Cn1 , (n + 1)Cn2 , . . . , (n + 1)Cnn



1 2 3 n+1

= lcm Cn+1 , 2Cn+1 , 3Cn+1 , . . . , (n + 1)Cn+1 .

Từ đây suy ra B | A. Vì Cnk | n! nên Cnk chỉ có ước nguyên tố p ≤ n. Do đó, (n + 1)Cnk chỉ có
ước nguyên tố p ≤ n + 1, ∀k = 0, n + 1. Với mỗi k = 1, n + 1, gọi p là ước nguyên tố của kCn+1
k

mà p ≤ n + 1. Đặt s = vp (B), thì ps ≤ n + 1 < ps+1. Vì [x + y] − [x] − [y] ≤ 1 nên


s      
k
 X n+1 k n+1−k
vp Cn+1 = vp ((n + 1)!) − vp (k!) − vp ((n + 1 − k)!) = − i −
i=1
pi p pi
vp (k)  
    s      
X n+1 k n+1−k X n+1 k n+1−k
= − i − + − i −
i=1
pi p pi pi p pi
i=vp (k)+1

≤ s − vp (k).
 
Do đó, vp kCn+1
k k
= vp (k) + vp Cn+1 ≤ s, ∀k = 1, n + 1. Điều này có nghĩa là A | B. Vậy
A = B và ta có điều phải chứng minh.
Ví dụ Q16. Cho số nguyên n > 1. Với mỗi tập con S khác rỗng của {1, 2, 3, . . . , n} , đặt
π(S) = x. Chứng minh rằng nếu k là số nguyên dương nhỏ hơn n, thì
x∈S

n   
Y n
lcm 1, 2, . . . , = gcd (π(S) : |S| = n + 1 − k) . (1)
j=k
j

Giải. Rõ ràng nếu các vế của đẳng thức trong đề bài mà có ước nguyên tố p, thì p ≤ n. Với
p (p ≤ n) là một số nguyên tố bất kì. Ta xét các trường hợp có thể xảy ra như sau
10 Phạm Xuân Thịnh

hni n
    
n n
i) Nếu k > , thì = 1. Do có đúng số nguyên dương chia hết
Q
lcm 1, 2, . . . ,
2 j=k j p
 
n
cho p và không vượt quá n nên trong tập {1, 2, 3, . . . , n} có đúng n − số không chia
p
hết cho p. Vì  
hni n
n+1−k ≤n− ≤n−
2 p
nên ta chọn được tập con S của {1, 2, 3, . . . , n} gồm n + 1 − k phần tử mà không có phần
tử nào chia hết cho p, tức là vp π(S) = 0. Do đó, vp (gcd (π(S) : |S| = n + 1 − k)) = 0,
tức là gcd (π(S) : |S| = n + 1 − k) = 1. Vậy (1) đúng.
ii) Nếu k = 1, thì (1) trở thành
n   
Y n
lcm 1, 2, . . . , = n!. (2)
j=1
j

Với p (p ≤ n) là một số nguyên tố bất kì, thì


..
      
n n n
lcm 1, 2, . . . , . p ⇐⇒ p ≤ ⇐⇒ j ≤ .
j j p
..
    
n n
Do đó vế phải của (2) chứa đúng nhân tử chia hết cho p. Nếu lcm 1, 2, . . . , . p,
p j
thì
   h i 
    n
n j
vp lcm 1, 2, . . . , = 1 + vp lcm 1; 2; . . . ;  
j p
   h i 
n
p
= 1 + vp lcm 1; 2; . . . ;   .
j

Khi ấy, ta có
  h i 
[Yp]
n 
n   !   n
Y n n p
vp lcm 1, 2, . . . , = + vp  lcm 1; 2; . . . ; .
   
j=1
j p j=1
j

  h i 
[Q n
p]
n  
p n
Lại do trong tích lcm 1; 2; . . . ;   có đúng số chia hết cho p nên
j=1 j p2
  h i  h n i  h i 
[Yn
]
 
p n   p 2 n
p n Y p2
vp  lcm 1; 2; . . . ;   = + vp  lcm 1; 2; . . . ;  .
   
j p 2 j
j=1 j=1

Suy ra
h n i   h i 
n    !     p 2 n
Y n n n Y p2
vp lcm 1, 2, . . . , = + 2 + vp  lcm 1; 2; . . . ;  .

j=1
j p p j=1
j
Cấp của số nguyên, căn nguyên thủy và ứng dụng 11

Cứ tiếp túc như vậy, ta được


n   ! X+∞  
Y n n
vp lcm 1, 2, . . . , = = vp (n!).
j=1
j i=1
pi

Vậy (2) được chứng minh.


hni
iii) Nếu 1 < k ≤ , thì từ (2) suy ra
2
n   ! k−1   !
Y n Y n
vp lcm 1, 2, . . . , = vp (n!) − vp lcm 1, 2, . . . , .
j=k
j j=1
j

Mặt khác, ta lại có

vp (gcd (π(S) : |S| = n + 1 − k)) = vp (n!) − max vp (π(S)) .


|S|=k−1

Do đó, ta chỉ cần chứng minh

k−1   !
Y n
vp lcm 1, 2, . . . , = max vp (π(S)) . (3)
j=1
j |S|=k−1

Ta sẽ tính max vp (π(S)) . Muốn vậy, ta chỉ việc chọn ra k − 1 số mà số mũ của p trong
|S|=k−1
chúng lớn nhất có thể. Gọi u là số nguyên dương lớn nhất sao cho pu ≤ n < pu+1 . Xảy ra
các trường hợp sau
 
n
• Nếu k−1 ≤ u , thì xét tập S gồm k−1 số chia hết cho pu trong tập {1; 2; 3; . . . ; n} .
p
Rõ ràng,

max vp (π(S)) = vp π(S) = (k − 1)u.
|S|=k−1
      
n n n
Để ý là k − 1 ≤ u ⇐⇒ p ≤ u
, tức là tất các các số lcm 1, 2, . . . ,
p k−1 j
với j = 1, k − 1 đều chia hết cho p . Thành thử, ta có
u

k−1   !
Y n
vp lcm 1, 2, . . . , = (k − 1).u
j=1
j

Vậy (3) đúng trong trường hợp này.


    
n n n
• Nếu tồn tại số nguyên t ≥ 0, 0 < v ≤ u−t−1 sao cho k −1 = u +· · ·+ u−t +
p   p  p 
n n
v, thì ta chọn tập con S của {1; 2; 3; . . . ; n} gồm u số chia hết cho pu , u−1 số
  p p
n
chia hết cho pu−1 , . . . , u−t số chia hết cho pu−t và x số chia hết cho pu−t−1 . Vì
p
12 Phạm Xuân Thịnh

   
n n
trong tập {1; 2; 3; . . . ; n} có đúng − số m mà vp (m) = u − i nên
pu−i pu−i+1
     
 n n n
max vp (π(S)) = vp π(S) = u. u + (u − 1) u−1
− u
|S|=k−1 p p p
   
n n
+ · · · + (u − t). − + v.(u − t − 1).
pu−t pu−t+1

Mặt khác ta dễ dàng thấy rằng


 
n


u nếu 1 ≤ j ≤ u
p 


   
n n


nếu


u−1 < j ≤ u−1
pu p
    

n 
vp lcm 1, 2, . . . , = ...
j    
n n


u−t nếu < j ≤ u−t


 u−t+1


  p   p 
n n


u − t − 1 nếu < j ≤ u−t + v


p u−t p

Do đó,
k−1   !      
Y n n n n
vp lcm 1, 2, . . . , = u. u + (u − 1) u−1
− u
j=1
j p p p
   
n n
+ · · · + (u − t). − u−t+1 + v.(u − t − 1)
pu−t p
= max vp (π(S)) .
|S|=k−1

Thành thử, trường hợp này (3) cũng đúng.

Vậy đẳng thức (1) được chứng minh.

Ví dụ 17 (IMOSL, 2007). Cho số nguyên k ≥ 2. Chứng minh rằng


 k k−1

v2 C22k+1 − C22k = 3k.

Giải. Ta có

k k−1 2k+1 ! 2k !
C22k+1 − C22k = −
(2k !)2 (2k−1 !)2
k−1 k−1
22 (2k + 1)(2k + 3) . . . (2k+1 − 1) 22 (2k − 1)(2k − 3) . . . 3.1
= −
" 2k−1! k−1
# 2 !
k−1
2Q k−1
k−1  2Q
22

2k + 2i − 1 − 2k − (2i − 1)
i=1 i=1
= .
2k−1!
Cấp của số nguyên, căn nguyên thủy và ứng dụng 13

k−1
2Q k−1
 2Q 
Đặt M = 2k + 2i − 1 − 2k − (2i − 1) , thì
i=1 i=1

 k k−1

v2 C22k+1 − C22k = 2k−1 + v2 (M) − v2 2k−1 ! = 1 + v2 (M).


k−1
2Q k−1
2Q
Dễ thấy f (x) = (x − (2i − 1)) là một hàm lẻ. Do đó, tồn tại đa thức g(x)
(x + 2i − 1) −
i=1 i=1 
với hệ số nguyên sao cho f (x) = x g(x) + Cx. Từ đây suy ra M = 23k g 2k + C.2k , với
3

 
k 1 1
 1
C = 2.1.3.5 . . . 2 − 1 1 + + + · · · + k .
3 5 2 −1

Ta có
k−1
2X 2X  k−1  2X k−1
1 1 1 k 1
2 = + k =2 k − 2i + 1)
.
i=1
2i − 1 i=1
2i − 1 2 − 2i + 1 i=1
(2i − 1) (2
 
Dễ thấy 1, 3, 5, . . . , 2k − 1 là một hệ thặng dư thu gọn (mod 2k ) nên với mỗi i ∈ 1, 3, 5, . . . , 2k − 1 ,

tập i, 3i, 5i, . . . , (2k − 1)i cũng là một hệ thặng dư thu gọn (mod 2k ). Do đó, với mỗi
 
i ∈ 1, 2, . . . , 2k−1 tồn tại duy nhất số ki ∈ 1, 2, . . . , 2k−1 sao cho (2i − 1).(2ki − 1) ≡ 1
(mod 2k ) hay 1 − (2i − 1).(2ki − 1) = Ai 2k . Khi ấy,
k−1
2X 2Xk−1   2Xk−1
1 1 2
= + (2ki − 1) − (2i − 1)2
i=1
(2i − 1) (2k − 2i + 1) i=1
(2i − 1) (2 k − 2i + 1)
i=1
k−1
2X 2k−1
2k (2i − 1) + 1 − (2i − 1)2 (2ki − 1)2 X
= k − 2i + 1)
− (2i − 1)2
i=1
(2i − 1) (2 i=1
2Xk−1 
2k .Mi 2k−1 4k − 1
= k − 2i + 1)
− .
i=1
(2i − 1) (2 3

Từ đây suy ra C = 22k−1 .A với A là một số nguyên lẻ nào đó. Thành thử,
 k k−1

v2 C22k+1 − C22k = 1 + v2 (M) = 1 + 2k − 1 + k = 3k.

Bài toán được giải quyết trọn vẹn.

Ví dụ 18 (Iran TST, 2012). Tìm tất cả các số nguyên n ≥ 2 sao cho với mọi k, l thỏa mãn
0 ≤ k, l ≤ n, ta có
Cnk + Cnl ≡ k + l (mod 2).

Giải. Từ giả thiết ta có n + 1 = Cn0 + Cn1 ≡ 1 (mod 2) suy ra n là số chẵn. Đặt n = 2t m, với
m là số lẻ và t ∈ N∗ . Nếu t ≥ 2, thì

n(n − 1)
Cn1 + Cn2 = n + = 2t m + 2t−1 m(2t m − 1) 6≡ 1 + 2 (mod 2).
2
14 Phạm Xuân Thịnh

Điều này mâu thuẫn với giả thiết. Do đó, t = 1 hay n = 2m. Gọi v là số nguyên dương sao cho
2v < n < 2v+1 và đặt n = 2v + u với 1 < u < 2v . Ta có
v
v2 Cn2 −2 = v2 (n!) − v2 ((2v − 2)!) − v2 ((n − 2v + 2)!)


= v2 ((2v + u)!) − v2 ((2v − 2)!) − v2 ((u + 2)!)


v  v   v   
X 2 +u 2 −2 u+2
= − −
i=1
2i 2i 2i
v h i  
X u u+2
= i
− i
+ v.
i=1
2 2
v −2 v
Mà C21v +u + Cn2 ≡ 1 (mod 2) và C21v +u = 2v + u làsố chẵn nên Cn2 −2 ≡ 1 (mod 2). Điều này
hui u+2
v
Cn2 −2 = 0. Chú ý là i −

có nghĩa là v2 ≥ −1, ∀i = 1, v. Do đó,
2 2i
v h i  
X u u+2
− + v ≥ 0.
i=1
2i 2i
hui 
u+2
Đẳng thức xảy ra khi và chỉ khi − = −1, ∀i = 1, v. Điều này chỉ xảy ra khi và
2i 2i
.
chỉ khi u + 2 .. 2v hay u = 2v − 2. Do đó, n = 2v+1 − 2.
Bây giờ với n = 2v+1 − 2, v ∈ N∗ , thì với mọi k thỏa mãn 0 ≤ k ≤ n, ta có

v2 C2kv+1 −2 = v2 (2v+1 − 2)! − v2 (k!) − v2 (2v+1 − 2 − k)!


  

+∞  v+1     v+1 
X 2 −2 k 2 −2−k
= − i −
i=1
2i 2 2i
+∞     
X k −k − 2
= −1 − i − .
i=1
2 2i

Xảy ra 2 trường hợp sau

i) Nếu k là số lẻ, thì do


   
k −k − 2 k−1 k+1
−1 − − = −1 − + + 1 = 1,
2 2 2 2

..
   
k −k − 2 
và −1 − i − ≥ 0, ∀i ≥ 2 nên v2 C k
2v+1 −2 ≥ 1 hay C k
2v+1 −2 . 2.
2 2i

ii) Nếu k là số chẵn, tức là k = 2t .m với m là số nguyên dương lẻ. Nếu t = 1, thì đặt
m + 1 = 2u .v. Khi đó, với i − 1 ≤ u, ta có
     
k −k − 2 m+1 m+1
−1 − − = −1 − i−1
− 1 + i−1 = 0,
2 2 2 2

còn với i − 1 > u, ta viết m = q.2i−1 + r, 0 < r < 2i−1 thì m + 1 = q.2i−1 + r + 1. Do
m + 1 không chia hết cho 2i−1 nên r + 1 cũng vậy. Do đó, r + 1 < 2i−1 . Từ đây suy ra
Cấp của số nguyên, căn nguyên thủy và ứng dụng 15

  h
m+1 m i
= i−1 . Chú ý là
2i−1 2
   
−m − 1 m+1 h m i
= − − 1 = − − 1,
2i−1 2i−1 2i−1
suy ra    
k −k − 2
−1 − − = 0.
2 2
Xét t ≥ 2.

• Nếu i ≤ t, thì
   
k −k − 2
= −1 − 2t−i .m − −2t−i .m − 1 = 0.

−1 − i − i
2 2

• Nếu i > t, thì do [−x] = −[x] − 1, ∀x ∈


/ Z nên
     t−1   t−1 
k −k − 2 2 .m + 1 2 .m
− i − i
= i−1
− + 1.
2 2 2 2i−1

Bây giờ ta đặt 2t−1 .m = q.2i−1 + r với 0 < r < 2i−1 , thì 2t−1 .m + 1 = q.2i−1 + r + 1.
Do 2t−1 .m + 1 không chia hết cho 2i−1 nênr + 1 cũng vậy.Vì thế, r + 1 < 2i−1 .
2t−1 .m + 1 r+1 2t−1 .m + 1 2t−1 .m
Khi ấy, từ i−1
= q + ta suy ra = . Thành thử,
  2  2i−1 2i−1 2i−1
k −k − 2
−1 − i − = 0.
2 2i

Vậy nếu k là số chẵn, thì ta luôn có v2 C2kv+1 −2 = 0, tức là C2kv+1 −2 không chia hết cho
2.

Từ những lập luận trên, ta suy ra C2kv+1 −2 ≡ k + 1 (mod 2), ∀k = 0, 2v+1 − 2. Theo đó, ta có

C2kv+1 −2 + C2kv+1 −2 ≡ k + l (mod 2).

Ví dụ 19 (Romania TST 2015). Cho trước số nguyên k ≥ 2. Với số nguyên n (n ≥ k) thay


đổi, trong tập {n − k + 1, n − k + 2, . . . , n} có nhiều nhất bao nhiêu số chia hết Cnk ?

Giải. Với mỗi n ≥ k, đặt f (n) là số các số trong tập {n − k + 1, n − k + 2, . . . , n} chia hết
Cnk . Vì
n! (n − k + 1)(n − k + 2) . . . (n − 1)n
Cnk = =
k!(n − k)! k!
nên nếu ta chọn n = k!, thì Cnk = (n − k + 1)(n − k + 2) . . . (n − 1), tức là tất cả các số
n − k + 1, n − k + 2, . . . , n − 1 đều chia hết Cnk . Do đó, max f (n) ≥ k − 1.
n≥k

Nếu max f (n) = k, thì tồn tại n ≥ k sao cho tất cả các số trong tập {n − k + 1, n − k + 2, . . . , n}
n≥k
đều chia hết Cnk . Điều đó có nghĩa là lcm (n − k + 1, n − k + 2, . . . , n) | Cnk . Để ý rằng n − k +
16 Phạm Xuân Thịnh

1, n−k + 2, . . . , n là k số nguyên liên tiếp nên trong số chúng có đúng một số chia hết cho k. Gọi
p là ước nguyên
 tốnàođócủa k và đặt vp (lcm
 (n − k +1, n − k + 2, . . . , n)) = s, thì với t > s,
n−k n n−k n n−k
ta phải có t
= t . Thật vậy, nếu t
< t , thì bằng cách đặt m = ,
p p p p pt
ta suy ra m.pt < n − k < (m + 1)pt < n, tức là trong dãy n − k + 1, n − k + 2, . . . , n phải có
một số chia hết cho pt . Vô lí ! Do đó, từ
           
n k n−k n k n−k
− i − = 0 nếu i = 1 hoặc i > s và i − i − ≤ 1, ∀2 ≤ i ≤ s
pi p pi p p pi
suy ra
+∞      
X n k n−k
Cnk

vp = vp (n!) − vp (k!) − vp ((n − k)!) = i
− i −
i=1
p p pi
      X s      
n k n−k n k n−k
= − − + − i −
p p p i=2
pi p pi
X  n   k   n − k  X s      
n k n−k
+ − i − = − i −
i>s
pi p pi i=2
pi p pi
≤ s − 1.

Điều này là vô lí, vì vp (lcm (n − k + 1, n − k + 2, . . . , n)) ≤ vp Cnk . Thành thử, max f (n) < k.
n≥k
Vậy max f (n) = k − 1.
n≥k

Ví dụ 20 (Romania TST 2016). Cho số nguyên n ≥ 3, n 6= 4 và tập

A = {2n − 1, 3n − 1, . . . , (n − 1)n − 1} .

Giả sử tất cả các phần tử của tập A đều không chia hết cho n.

a) Chứng minh rằng n không chia hết cho bình phương của một số nguyên tố.

b) Liệu n có bắt buộc phải là số nguyên tố hay không?

Giải.
n
a) Giả sử ngược lại là tồn tại số nguyên tố p sao cho p2 | n. Đặt m = + 1, thì 2 < m < n − 1,
p
n
tức là m − 1 ∈ A. Rõ ràng, = m − 1 | m − 1 suy ra mọi ước nguyên tố q (q 6= p) của
n n

  p
n
n ta có vq (n) = vq ≤ vq (mn − 1) .
p

• Nếu p 6= 2, thì do p | m − 1 nên theo bổ đề LTE, ta có

vp (mn − 1) = vp (m − 1) + vp (n) > vp (n).

• Nếu p = 2, thì n là số chẵn và 2 | m − 1 nên

v2 (mn − 1) = v2 (m − 1) + v2 (m + 1) + v2 (n) − 1 > v2 (n).


Cấp của số nguyên, căn nguyên thủy và ứng dụng 17

Thành thử, với mọi ước nguyên tố q của n, ta đều có vq (n) ≤ vq (mn − 1) hay n | mn − 1.
Điều mâu thuẫn này chứng tỏ giả sử trên của ta là sai. Vậy n không thể chia hết cho
bình phương của bất kì số nguyên tố nào.

Ví dụ 21 (IMOSL, 2008). Cho số nguyên n > 1. Chứng minh rằng tập các số dư khi chia các
n−1
số C20n −1 , C21n −1 , C22n −1 , . . . , C22n −1−1 cho 2n là {1; 3; . . . ; 2n − 1} .

Giải. Dễ thấy yêu cầu của bài toán đã cho tương đương với việc chứng minh
n n−1
o
C20n −1 , C21n −1 , C22n −1 , . . . , C22n −1−1

là một hệ thặng dư thu gọn (mod 2n ). Công việc đó được chia thành 2 việc nhỏ sau

i) Chứng minh gcd 2, C2kn −1 = 1, ∀k = 1, 2n−1 − 1.



(1)
Sử dụng công thức Legendre, ta có

v2 C2kn −1 = v2 ((2n − 1)!) − v2 (k!) − v2 ((2n − 1 − k)!)




n−1  n     n 
X 2 −1 k 2 −1−k
= − i −
i=1
2i 2 2i
n−1     
X k −k − 1
= − i − − (n − 1).
i=1
2 2i

Với mỗi i = 1, n − 1 đặt k = 2i .ui + ri , 0 ≤ ri < 2i , thì


     
k −k − 1 −ri − 1
− i − =− = 1.
2 2i 2i

Suy ra v2 C2kn −1 = 0.

ii) Chứng minh C2kn −1 6≡ C2l n −1 (mod 2n ), ∀k, l = 1, 2n−1 − 1, k > l. (2)

Để chứng minh (2) ta chỉ cần chứng minh v2 C2kn −1 − C2l n −1 < n. Ta có

(2n − l − 1)(2n − l − 2) . . . (2n − k) − (l + 1)(l + 2) . . . k


C2kn −1 − C2l n −1 = C2kn −1 .
(2n − l − 1)(2n − l − 2) . . . (2n − k)

Chú ý. Việc chứng minh ii) có thể tiến hành theo cách đơn giản hơn bằng cách sử dụng kết
quả: nếu p là một số nguyên tố và k, n là các số tự nhiên với k = 0, pn − 1, ta có

vp Cpkn = n − vp (k).


Bây giờ với k, l = 1, 2n−1 − 1, k > l đặt M = C2kn −1 − C2l n −1 . Xảy ra các trường hợp sau
18 Phạm Xuân Thịnh

• Nếu k − l là số chẵn, thì

M = C2kn − C2k−1
n + · · · + C2l+2 l+1
n − C2n .

Vì v2 (C2i n ) = n−v2 (i) nên nếu i là số lẻ, thì v2 (C2i n ) = n. Xét các số i chẵn và i = l + 1, k,
thì v2 (C2i n ) < n. Đặt A là tập các số chẵn từ l + 1 đến k và s = min v2 (C2i n ) . Nếu tồn
 i∈A
tại a, b ∈ A (a < b) sao cho v2 (C2an ) = v2 C2bn = s, thì v2 (a) = v2 (b) = n − s suy ra a =
.
2n−s .u, b = 2n−s .v với u, v là các số nguyên dương lẻ. Khi ấy, b − a = 2n−s (v − u) .. 2n−s+1.
Do đó tồn tại số c nằm giữa a và b sao cho v2 (c) ≥ n − s + 1 hay v2 (C2cn ) ≤ s − 1 < s.
Mâu thuẫn ! Do đó chỉ có đúng một a ∈ A sao cho v2 (C2an ) = s hay v2 (M) = s < n.

• Nếu k − l là số lẻ, thì

M = C2kn − C2k−1
n + · · · + C2l+1 l
n − 2C2n −1 .


Rõ ràng v2 2C2l n −1 = 1. Mặt khác v2 (C2i n ) = n−v2 (i) ≥ 2, ∀i ∈ A và v2 (C2i n ) = n, ∀i ∈
{l + 1, l + 2, . . . , k} \ A nên v2 (M) = 1.

Vậy trong mọi trường hợp ta đều có v2 (M) < n, tức là C2kn −1 6≡ C2l n −1 (mod 2n ), ∀k, l =
1, 2n−1 − 1, k > l.
Ví dụ 22.

Giải.

Ví dụ 23.

Giải.

Ví dụ 24.

Giải.

Ví dụ 25.

Giải.

Ví dụ 26.

Giải.

Ví dụ 27.
Cấp của số nguyên, căn nguyên thủy và ứng dụng 19

Giải.

Ví dụ 28.

Giải.

Tài liệu
[1] Hà Huy Khoái: Số học thuật toán.

[2] Hà Duy Hưng: Extending an IMO Problem, Mathematical Excalibur.

[3] Nguyễn Duy Liên: Chuyên đề: Cấp số nguyên, căn nguyên thủy và ứng dụng.

[4] Amir Hossein Parvardi: Lifting The Exponent Lemma.

[5] Moshe Roitman: On Zsigmondy Primes.

[6] Titu Andresscu: Problems from the book.

[7] Các nguồn tài liệu từ Internet: http://www.mathscope.org;


http://www.artofproblemsolving.com

You might also like